Buscar

Calculo I - 20 provas ufmg

Esta é uma pré-visualização de arquivo. Entre para ver o arquivo original

prova2- 2014.1-solucao -Aldo.pdf
CÁLCULO I - prova 2 - GABARITO
Questão 1
Estude a função (achando domı́nio, máximos, mı́nimos, pontos de
inflexão e asśıntotas) e esboçe seu gráfico.
f (x) =
3
√
x2ex
SOLUÇÃO
1) Domı́nio:
D = R
2) Simmetrias:
f (−x) = 3
√
(−x)2e−x = 3
√
x2e−x
f (−x) 6= f (x)
f (−x) 6= −f (x)
f(x) não é nem par nem impar.
3) Interseções com os eixos cartesianos:
f(x) = 0 ⇒ x = 0
Logo o gráfico da f corta os eixos X e Y só na origem O = (0, 0).
4) Sinal da f :
f (x) ≥ 0 ∀x ∈ R
5) Limites:
lim
x→+∞
f (x) = lim
x→+∞
x2/3ex = +∞
e em busca de um eventual assintota obliqua calculamos
lim
x→+∞
f (x)
x
= lim
x→+∞
ex
x1/3
= 3 lim
x→+∞
ex
x−2/3
= 3 lim
x→+∞
x2/3ex = +∞
Logo para x → +∞ temos que f(x) → +∞ e não tem asśıntota
obliqua.
lim
x→−∞
f (x) = lim
x→−∞
x2/3ex = lim
y→+∞
y2/3
ey
=
2
3
lim
y→+∞
y−1/3
ey
= 0
Logo y = 0 é asśıntota horizontal para f quandi x→ −∞.
6) Derivadas:
f (x) = x
2
3 · ex
f ′ (x) =
2
3
x−
1
3 · ex + x
2
3 · ex = ex ·
(
2
3
x−
1
3 + x
2
3
)
= ex
2 + 3x
3x1/3
Obs: a derivada f ′(x) não é definida em x = 0.
f ′ (x) ≥ 0 ⇒ 2
3
x−
1
3 + x
2
3 ⇒ 2 + 3x
3x1/3
≥ 0
Logo
f ′ (x) > 0 ⇒ x ∈ (−∞,−2/3) ∪ (0,+∞)
e
f ′ (x) < 0 ⇒ x ∈ (−2/3, 0)
e em x = −2/3 temos que a derivada zera i.e. f ′ (−2/3) = 0 e em
x = 0 a derivada não existe.
Logo para x < −2/3 e para x > 0 a função é crescente. Entre −2/3
e 0 a função é decrescente. A derivada se anula no ponto x = −2/3
e não existe no ponto x = 0. Logo
em x = −2/3 a função tem um máximo sendo f(−2/3) = (4/9)1/3 ·
e−2/3 ≈ 0.39:
o máximo ocorre então no ponto (−2/3, (4/9)1/3e−2/3)
e, dado que a função decrescente a esquerda de x = 0 e crescente a
direita de x = 0,
em x = 0 a função tem um mı́nimo, sendo f(0) = 0:
o mı́nimo ocorre então no ponto (0, 0).
Em x = 0 a função de decrescente vira crescente, mas a derivada
não existe em x = 0. Para entender o comportamento da derivada
na vizinhança de x = 0 calculamos:
lim
x→0−
f ′ (x) = lim
x→0−
ex
2 + 3x
3x1/3
=
2
0−
= −∞
lim
x→0+
f ′ (x) = lim
x→0+
ex
2 + 3x
3x1/3
=
2
0+
= +∞
logo (0; 0) è una cúspide.
Derivada segunda:
f ′′ (x) = ex ·
(
4
3
x−
1
3 + x
2
3 − 2
9
x−
4
3
)
=
ex
9x
4
3
·
(
12x+ 9x2 − 2
)
f ′′ (x) = 0 ⇒ 9x2+12x−2 = 0 ⇒ x = −12±
√
216
18
=
−2±
√
6
3
f ′′ (x) > 0 ⇒ 12x+ 9x2 − 2 > 0 ⇒
⇒ x ∈
(
−2 +
√
6
3
,+∞
)⋃(
−∞, −2−
√
6
3
)
Logo a função é convexa se x > −2+
√
6
3 e se x <
−2−
√
6
3 e é concava
no intervalo (−2−
√
6
3 ,
−2+
√
6
3 ) (em x = 0 f
′′ non existe) tendo então 2
pontos de inflexão, a saber:
xF1 =
−2−
√
6
3
≈ −1, 48 xF2 =
−2 +
√
6
3
≈ 0, 15
Veja o gráfico aqui em baixo
Questão 2
Determine o valor da área A da região do plano entre a parábola
y = 3− x2 e a reta y = x+ 1.
SOLUÇÃO
Reta e parábola se cruzam em dois pontos cujas abscissas são as
soluções da equação
3− x2 = x+ 1 ⇒ x2 + x− 2 = 0 ⇒ x = −2 e x = 1
Veja um esboço da região do plano em questão.
A área é dada então pela integral definida∫ 1
−2
[
(3− x2)− (x+ 1)
]
dx =
∫ 1
−2
[
2− x2 − x
]
dx =
=
(
2x− x
3
3
− x
2
2
)∣∣∣∣1
−2
=
(
2− 1
3
− 1
2
)
−
(
−4 + 8
3
− 2
)
=
9
2
Questão 3
Encontre o máximo absoluto e o mı́nimo absoluto da função
f (x) =
1
2
ln(1 + x2) + arctan(x)− x
no intervalo [−1,
√
3]
Lembrete
0, 693 ≤ ln 2 ≤ 0, 694 3.141 ≤ π ≤ 3.142 1.732 ≤
√
3 ≤ 1.733
Solução
Derivada:
f ′(x) =
x
1 + x2
+
1
1 + x2
− 1 = x− x
2
1 + x2
e f ′(x) = 0 ⇒ x = 1, x = 0. Ambos os pontos cŕıticos x = 0 e
x = 1 estão no intervalo (−1,
√
3).
Calculando a função nos pontos cŕıticos x = 0 e x = 1. Temos
f(0) =
1
2
ln 1 + arctan(0) = 0
f(1) =
1
2
ln 2 +
π
4
− 1 = 1
2
[ln 2 + π/2− 2] ≈ 0, 13
Calculando a função nos extremos. temos
f(−1) = 1
2
ln 2− π
4
+ 1 =
1
2
[ln 2− π/2 + 2] ≈ 0, 56
f(
√
3) =
1
2
ln 4 +
π
3
−
√
3 = ln 2 +
π
3
−
√
3 ≈ 0, 008
Logo
1
2 [ln 2− π/2 + 2] é o máximo absoluto e ocorre em x = −1
e
0 é o mı́nimo absoluto que ocorre em x = 0
prova3- 2014.1 - solu??o - Aldo.pdf
CÁLCULO I - PROVA 3 - Turma D
1. Calcule a integral definida
I =
∫ √3/2
0
x3√
1− x2
dx
Solução: com a substituição x = sen θ temos que dx = cos θdθ
sendo que x = 0 corresponde a θ = 0 e x =
√
3/2 corresponde
a sen θ =
√
3
2 , i.e. θ = π/3. Isto é, se x varia no intervalo
x ∈ [0,
√
3
2 ] então θ varia no intervalo θ ∈ [0,
π
3 ].
Observe que nesse intervalo cos θ ≥ 0 e portanto
√
1− sen 2θ =√
cos2 θ = | cos θ| = cos θ.
Logo a integral vira
I =
∫ √3/2
0
x3√
1− x2
dx =
∫ π/3
0
sen 3θ cos θ√
1− sen 2θ
dθ =
∫ π/3
0
sen 3θdθ =
=
∫ π/3
0
(1− cos2 θ) sen θdθ
Pondo agora u = cos θ e logo du = − sen θdθ, sendo que θ = 0
corresponde a u = 1 e θ = π/3 a u = 12 , a integral vira
I =
∫ π/3
0
(1− cos2 θ) sen θdθ = −
∫ 1/2
1
(1−u2)du =
∫ 1
1/2
(1−u2)du =
=
[
u− u
3
3
]1
1/2
=
(
1− 1
3
)
−
(1
2
− 1
24
)
=
5
24
11 pontos
2. Calcule a integral imprópria
I =
∫ ∞
0
2
(x2 + 1)(x+ 1)
dx
Solução Reescrevendo o integrando em frações parcias temos
2
(x2 + 1)(x+ 1)
=
1
x+ 1
− x
x2 + 1
+
1
x2 + 1
Logo
I =
∫ ∞
0
2
(x2 + 1)(x+ 1)
dx = lim
K→∞
∫ K
0
[
1
x+ 1
− x
x2 + 1
+
1
x2 + 1
]
dx
= lim
K→∞
[
ln(x+ 1)− 1
2
ln(x2 + 1) + arctan x
]K
0
=
= lim
K→∞
[
ln
x+ 1√
x2 + 1
+ arctanx
]K
0
=
lim
K→∞
[
ln
[
K + 1√
1 +K2
]
+ arctanK
]
=
lim
K→∞
ln
[
K + 1√
1 +K2
]
+ lim
K→∞
arctanK =
ln
 lim
K→∞
(1 + 1K )√
1 + 1K2
+ lim
K→∞
arctanK = ln 1 +
π
2
=
π
2
12 pontos
3. Calcule o volume do sólido obtido pela rotação da região do
primeiro quadrante limitada pelas curvas
y = e
√
x, x = 0, , x = 1
em torno do eixo x.
Solução. Veja abaixo um esboço da região do plano a ser
rodada emvolta do eixo x.
Integrando por fatias ciĺındricas o volume pedido é
V =
∫ 1
0
π(e
√
x)2dx = π
∫ 1
0
(e2
√
x)dx
Fazendo a mudança de variáveis u = 2
√
x e logo du = dx√
x
e
logo dx = 12udu com u variando de 0 a 2, temos
V = π
∫ 1
0
(e2
√
x)dx =
π
2
∫ 2
0
ueudu =
π
2
[
ueu − eu
]2
0
=
π
2
[
2e2 − e2 − (0− 1)
]
=
π
2
[
e2 + 1
]
11 pontos
3? Prova - 2008.pdf
Universidade Federal de Minas Gerais
Instituto de Ciências Exatas – ICEx
Departamento de Matemática
Cálculo I
1o Semestre de 2008 – Resolução da 3a Prova - 25/06/2008
1a questão. (7 pontos) Calcule a área limitada pelas curvas x = y3 − y e x = 3y.
Solução. Pontos de interseção: y3 − y = 3y ⇔ y3 − 4y = 0 ⇔ y(y2 − 4) = 0
⇔ y = −2, y = 0 ou y = 2.
Colocando x em função de y as funções são: x = f(y) = 3y, x = g(y) = y3 − y e a
área pedida é:
A =
∫
0
−2
(g(y) − f(y)) dy +
∫
2
0
(f(y)− g(y)) dy
A =
∫
0
−2
(y3 − y − 3y) dy +
∫
2
0
(3y − y3 + y) dy
A =
∫
0
−2
(y3 − 4y) dy +
∫
2
0
(4y − y3) dy
A =
(
y4
4
− 2y2)|0
−2
+ (
−y4
4
+ 2y2
)∣
∣
∣
∣
2
0
A =
(
−2
4
4
+ 2.4) − ((−2)
4
4
− 2.4
)
A = −4 + 8 − 4 + 8 = 8.
2a questão. (10 pontos) Ache o volume gerado pela rotação da região limitada pelas curvas y =
4x − x2, y = 8x − 2x2, ao redor de x = −2.
Solução. Pontos de interseção das duas curvas:
−2x2 + 8x = −x2 + 4x ⇔ −x2 + 4x = 0 ⇔ x = 0, x = 4.
Por cascas ciĺındricas, a integral será de 0 até 4. Acima do ponto de abscissa x, temos:
Raio da casca= x+2, f(x) = 8x−2x2, g(x) = 4x−x2. Altura da casca = f(x)−g(x).
V =
∫
4
0
2π(x + 2)[(8x − 2x2) − (4x − x2)] dx.
V = 2π
∫
4
0
(x + 2)(8x − 2x2 − 4x + x2) dx.
V = 2π
∫
4
0
(x + 2)(−x2 + 4x) dx ⇒ V = 2π
∫
4
0
(−x3 + 4x2 − 2x2 + 8x) dx
V = 2π
∫
4
0
(−x3 + 2x2 + 8x) dx ⇒ V = 2π(−x
4
4
+ 2
x3
3
+ 4x2)|4
0
V = 2π(−4
4
4
+
2.43
3
+ 4.(16))
V = 2π(−64 + 264
3
+ 64) ⇒ V = 256
3
π.
1. (16 pontos) Calcule
(a) I =
∫
1
(4 + x2)2
dx.
Solução . x = 2tgθ, dx = 2 sec2 θ dθ.
I =
∫
1
(4 + 4tg2θ)2
2 sec2 θ dθ =
∫
1
16(1 + tg2θ)2
2 sec2 θ dθ
=
1
8
∫
sec2 θ
sec4 θ
dθ =
1
8
∫
1
sec2 θ
dθ =
1
8
∫
cos2 θ dθ
=
1
8
∫
1 + cos 2θ
2
dθ =
1
16
(θ +
sen2θ
2
) + C =
1
16
θ +
1
16
senθ cos θ + C. Como
tgθ =
x
2
, então senθ =
x√
4 + x2
, cos θ =
2√
4 + x2
, e
I =
∫
1
(4 + x2)2
dx =
1
16
arctg
x
2
+
1
16
2x
4 + x2
+ C
I =
1
16
arctg
x
2
+
1
8
x
4 + x2
+ C
(b) I =
∫
∞
1
x2e−3x dx.
Solução . Por partes, u = x2, dv = e−3x, du = 2xdx, v =
−e−3x
3
∫
x2e−3x dx =
−x2e−3x
3
+
∫
2x
3
e−3x dx.
De novo por partes, U =
2x
3
, dV = e−3x, dU =
2
3
dx, V =
−e−3x
3
, e
∫
2x
3
e−3x dx =
−2x
9
e−3x+
2
9
∫
e−3x dx =
−2x
9
e−3x − 2
27
e−3x
Substituindo na 1a integral
∫
x2e−3x dx =
−x2e−3x
3
+
−2x
9
e−3x − 2
27
e−3x.
Portanto
I = lim
t→∞
∫ t
1
x2e−3x dx = lim
t→∞
(−x2e−3x
3
+
−2x
9
e−3x − 2
27
e−3x
)∣
∣
∣
∣
t
1
I = lim
t→∞
( −t2
3e3t
+
−2t
9e3t
︸ ︷︷ ︸
L’Hôpital
− 2
27e3t
)
−
(−1
3e3
− 2
9e3
− 2
27e3
)
I = lim
t→∞
( −2t
9e3t
︸︷︷︸
L’Hôpital
+
−1
27e3t
− 2
27e3t
)
−
(−1
3e3
− 2
9e3
− 2
27e3
)
I = ( 0 + 0 + 0 ) −
(−1
3e3
− 2
9e3
− 2
27e3
)
⇒ I = 17
27e3
(c) I =
∫
1 − x + 2x2 − x3
x(x2 + 1)2
dx.
Solução . Por frações parciais:
1 − x + 2x2 − x3
x(x2 + 1)2
=
A
x
+
Bx + C
x2 + 1
+
Dx + E
(x2 + 1)2
−x3 + 2x2 − x + 1 = A(x2 + 1)2 + (Bx + C)x(x2 + 1) + (Dx + E)x
x4 : A + B = 0 A = 1
x3 : C = −1 B = −1
x2 : 2A + B + D = 2 =⇒ C = −1
x : C + E = −1 D = 1
1 : A = 1 E = 0
Voltando à integral:
I =
∫ (
1
x
+
(−)x + (−1)
x2 + 1
+
x
(x2 + 1)2
dx
I =
∫
1
x
dx +
∫ −1
x2 + 1
dx +
∫ −x
x2 + 1
dx +
∫
x
(x2 + 1)2
dx
I = ln|x| − 1
2
ln(x2 + 1) − arctgx − 1
2
1
x2 + 1
onde as duas últimas integrais saem pela substituição u = x2 + 1, du =
2xdx.
(d) I =
∫
3
1/3
√
x
x2 + x
dx. (Sugestão: Faça u =
√
x.)
Solução . u =
√
x ⇒ u2 = x ⇒ 2udu = dx. Logo
∫ √
x
x2 + x
dx =
∫
u
u4 + u2
2udu =
∫
2
1
u2 + 1
du = 2 arctanu = 2 arctan
√
x. Donde
I = 2 arctan
√
x
∣
∣
∣
∣
3
1/3
= 2
(
π
3
− π
6
)
⇒ I = π
3
.
exame especial 2014.1-solucao - Aldo.pdf
CÁLCULO I - EXAME ESPECIAL
1. Considere a região R do plano delimitada pelo gráfico da função
f(x) = (4x− x2 − 3)1/4
e pelo eixo x. Calcule o volume do sólido de rotação obtido girando
a região R em torno do eixo x.
Solução
O dóminio Df de definição da f(x) é o conjunto dos pontos x ∈ R
tais que
4x− x2 − 3 ≥ 0 =⇒ x ∈ [1, 3]
Observe que f(1) = f(3) = 0 e que f(x) > 0 no intevalo aberto
(1, 3). Além disso
f ′(x) =
1
4
(4x− x2 − 3)3/4(4− 2x) =⇒ f ′(x) = 0 quando x = 2
i.e. f(x) tem um ponto de max em x = 2.
Veja um esboço da região R entre o gráfico da f(x) e o eixo x em
baixo.
O volume do sólido de rotação obtido girando a região R em torno
do eixo x se encontra integrando por fatias ciĺındricas
V =
∫ 3
1
π[f(x)]2dx = π
∫ 3
1
√
4x− x2−dx = π
∫ 3
1
√
1− (x− 2)2dx
tendo observado que
4x− x2 − 3 = 1− (x2 − 4x+ 4) = 1− (x− 2)2
Fazendo a mudança u = x− 2, du = dx, e x ∈ [1, 3]⇒ u ∈ [−1, 1]
V = π
∫ 1
−1
√
1− u2du = 2π
∫ 1
0
√
1− u2du
Fazendo a mudança u = sen θ, du = cos θdθ, e u ∈ [0, 1] ⇒ θ ∈
[0, π/2] e usando a identidade cos θ = 12 [1 + cos(2θ)] obtemos
V = 2π
∫ π
2
0
cos2 θdθ = π
∫ π
2
0
(1+cos(2θ)dθ = π
[
θ +
sen (2θ)
2
]π
2
0
=
π2
2
2. Calcule os seguintes limites
lim
x→+∞
e
√
lnx
√
x
, lim
x→0
√
2x3 + 1−
√
cosx
x2
Solução.
• limite 1
Fazendo a mudança y =
√
lnx e logo x = ey
2
,
sendo que x→∞⇒ y →∞
lim
x→+∞
e
√
lnx
√
x
= lim
y→+∞
ey√
ey2
= lim
y→+∞
ey
ey2/2
= lim
y→+∞
1
ey2/2−y
=
1
limy→+∞ ey
2[1/2−1/y] =
1
elimy→+∞(y
2[1/2−1/y] =
1
e(∞)
2[ 12−
1
∞ ]
=
1
∞
= 0
OBS: para esse limite l’Hopital é inconclusivo.
• limite 2
Usando l’Hopital e o limite notável limx→0
senx
x = 1
lim
x→0
√
2x3 + 1−
√
cosx
x2
= lim
x→0
6x2√
2x3+1
+ senx
2
√
cosx
2x
=
= lim
x→0
3x√
2x3 + 1
+
senx
4x
√
cosx
= 3 lim
x→0
3x√
2x3 + 1
+
1
4
lim
x→0
senx
x
lim
x→0
1√
cosx
=
= 3
0√
0 + 1
+
1
4
· 1 · 1
1
=
1
4
Alternativamente, usando o limite notável limx→0
(1−cosx)
x2 =
1
2
lim
x→0
√
2x3 + 1−
√
cosx
x2
= lim
x→0
2x3 + 1− cosx
x2(
√
2x3 + 1 +
√
cosx)
=
lim
x→0
2x
(
√
2x3 + 1 +
√
cosx)
+lim
x→0
(1− cosx)
x2
lim
x→0
1√
2x3 + 1 +
√
cosx
=
0√
1 +
√
cos 0
+
1
2
1√
1 +
√
cos 0
=
1
4
3. Estude a função
f(x) = ln(e2x − 4ex + 5)
e esboçe seu gráfico.
O domı́nio Df da f(x é o conjunto dos x tais que e
2x− 4ex + 5 > 0,
i.e. fazendo y = ex, y2 − 4y + 5 > 0 que é sempre verificada. Logo
Df = R.
Sendo f(x = 0) = ln 2, temos que a função cruza o eixo y no ponto
(0, ln 2).
Sendo f(x) = 0 ⇒ e2x − 4ex + 5 = 10 ⇒ e2x − 4ex + 4 = 0 ⇒
(ex− 2)2 = 0⇒ x = ln 2, temos que a função intercepta o eixo x no
ponto (ln 2, 0).
Observe ainda que
f(x) = ln(e2x − 4ex + 5) = ln[1 + (ex − 2)2]
Logo f(x) > 0 para todo x 6= ln 2 e f(x) = 0 para x = ln 2. Isto é,
o gráfico da f está todo acima do eixo x e o toca somente no ponto
(ln 2, 0) que é portanto o ponto de mı́nimo absoluto da f(x).
Procuramos eventuais asśıntotas. Temos
lim
x→+∞
f(x) = lim
x→+∞
ln[e2x−4ex+5] = lim
x→+∞
ln[e2x(1−4/ex+5/e2x)] =
= lim
x→∞
[
2x+ ln(1− 4/ex + 5/e2x)
]
= +∞+ ln(1− 4
∞
+
5
∞
) = +∞
lim
x→−∞
f(x) = lim
x→−∞
ln[e2x − 4ex + 5] = ln 5
Logof(x) tem asśıntota horizontal a reta y = ln 5 para x→ −∞.
Vamos checar se f(x) tem assintota obliquo quando x→ +∞
lim
x→∞
f(x)
x
= lim
x→∞
[
2x+ ln(1− 4/ex + 5/e2x)
]
x
= 2
e
lim
x→∞
f(x)− 2x = lim
x→∞
ln(1− 4/ex + 5/e2x) = 0
Logo f(x) tem assintota obliqua a reta y = 2x para x→ +∞.
Calculando as derivadas
f ′(x) =
2e2x − 4ex
e2x − 4ex + 5
f ′(x) = 0 =⇒ 2e2x − 4ex = 0 =⇒ ex − 2 = 0 =⇒ x = ln 2
Logo f(x) so tem um ponto cŕıtico em x = ln 2 que já sabemos ser
o ponto de mı́nimo absoluto da f . Observe ainda que f ′(x) < 0
quendo x < ln 2 e f ′(x) > 0 quando x > ln 2. Logo f(x) é decres-
cente no intervalo (−∞, ln 2) e crescente no intervalo (ln 2,+∞).
f ′′(x) =
(4e2x − 4ex)[e2x − 4ex + 5]− (2e2x − 4ex)2
[e2x − 4ex + 5]2
=
=
−4e3x + 20e2x − 20ex
[e2x − 4ex + 5]2
f ′′(x) = 0 =⇒ −4e3x+20e2x−20ex = 0 =⇒ −e2x+5ex−5 = 0 =⇒
=⇒ ex = 5±
√
5
2
=⇒ x = ln
[
5−
√
5
2
]
e x = ln
[
5 +
√
5
2
]
Observe ainda que
f ′(x) > 0 quando x ∈ (ln 5−
√
5
2 , ln
5+
√
5
2 )
f ′(x) < 0 quando x ∈ (−∞, ln 5−
√
5
2 ) ∪ (ln
5+
√
5
2 ,+∞)
Logo f(x) é convexa no intervalo x ∈ (ln 5−
√
5
2 , ln
5+
√
5
2 ), é cóncava
para x ∈ (−∞, ln 5−
√
5
2 ) ∪ (ln
5+
√
5
2 ,+∞) e tem 2 pontos de inflexão
em x = ln 5+
√
5
2 e x = ln
5−
√
5
2 .
Veja abaixo um esboço do gráfico
4. Ache os pontos de máximo absoluto e de mı́nimo absoluto da
função
f(x) = x3 − x+ 1
no intervalo fechado [−1, 2].
Achamos os pontos cŕıticos da f .
f ′(x) = 3x2 − 1
i.e.
f ′(x) = 0 =⇒ 3x2 − 1 = 0 =⇒ x = ± 1√
3
Logo, f(x) tem 2 pontos cŕıticos x = − 1√
3
e x = + 1√
3
, ambos
contidos dentro do intervalo (−1, 2).
Temos ainda que
f(−1/
√
3) = 1 +
2
3
√
3
f(1/
√
3) == 1− 2
3
√
3
f(−1) = 1
f(2) = 7
Logo o mı́nimo absoluto da f no intervalo [−1, 2] é 1 − 2
3
√
3
que é
atingido no ponto x = 1/
√
3.
O máximo absoluto da f no intervalo [−1, 2] é 7 que é atingido no
ponto x = 2
exemplo de prova 1 - guerassimov.pdf
Exemplo da 1a prova de Cálculo 1
Nota máxima: 33 pontos. É permitido o uso de livros e cadernos. É estritamente proibido
copiar
de livros, cadernos, trabalhos dos colegas. Para ter nota máxima deve-se explicar cada
passo do racioćınio e justificar toda conclusão.
1. Encontrar lim
x→1
3√x−1
4√x−1 .
2. Encontrar δ>0 tal que |tg x−
√
3| < 0, 1 para todo x tal que |x−π
3
|<δ.
3. Encontrar lim
x→a
aa
x−axa
ax−xa , onde a>1.
4. Encontrar a reta tangente à curva xy=16 no ponto (2, 4).
5. Para a função f(x) = 2arc tg x − arc sen x encontrar os intervalos de crescimento e decresci-
mento. Esboçar o gráfico.
exemplo de prova 2 -guerassimov.pdf
Exemplo da 2a prova de Cálculo 1
Nota máxima: 33 pontos. É permitido o uso de livros e cadernos. É estritamente proibido
copiar de livros, cadernos, trabalhos dos colegas. Para ter nota máxima deve-se explicar cada
passo do racioćınio e justificar toda conclusão.
1. Encontrar a derivada da função f(x) = (senx)arc senx.
2. Encontrar os pontos onde existe derivada f ′(x) para função f(x) = |(x−1)(x−2)2(x−3)3|.
3. Encontrar lim
x→1
(
1
lnx
− πctg(πx)
)
.
4. Analisar a função f(x) = (x−5) 3
√
x2 e esboçar o gráfico.
5. Nos lados de retângulo ABCD com |AB|=12 e |BC|=8 marcam-se pontos P∈[AB], Q∈[BC],
R∈[CD] de modo que os triângulos PBQ e PRQ são congruentes. Encontrar o valor mı́nimo de
|PQ|.
exemplo prova 3- guerassimov.pdf
Exemplo da 3a prova de Cálculo 1
Nota máxima: 34 pontos. É permitido o uso de livros e cadernos. É estritamente proibido
copiar de livros, cadernos, trabalhos dos colegas. Para ter nota máxima deve-se explicar cada
passo do racioćınio e justificar toda conclusão.
1. Calcular as integrais indefinidas: (a)
∫
dx
(x2−1)2 ; (b)
∫
dx
x
√
lnx
.
2. Calcular o comprimento do gráfico da função f(x) = ln(senx) no intervalo [0, π
3
].
3. Calcular a área da superf́ıcie obtida pela rotação da curva 9x = y2+18, 2 ≤ x ≤ 6 ao redor
do eixo x.
4. Calcular o volume do sólido obtido pela rotação da região limitada pelas curvas y=lnx, y=0,
x=e ao redor do eixo x.
Prova 1 - 2011-1.pdf
Prova 1 - 2013-1.pdf
Prova 1 - 2014.01 - solu??o - Aldo.pdf
1a PROVA CÁLCULO I: 15/03/2014
1. Calcule os seguintes ĺımites
a) lim
x→2
ln(x− 1)
x2 + x− 6
b) lim
x→0
ex
2 − cos(3x)
x2
Solução a): 1
5
lim
x→2
ln(x− 1)
x2 + x− 6
= lim
x→2
ln(x− 1)
(x+ 3)(x− 2)
= lim
x→2
ln[1 + (x− 2)]
(x+ 3)(x− 2)
=
= lim
x→2
1
(x+ 3)
lim
x→2
ln[1 + (x− 2)]
(x− 2)
=
1
5
lim
t→0
ln[1 + t]
t
=
1
5
Pois limt→0
ln[1+t]
t
= 1 é limite notável
Solução b): 11
2
lim
x→0
ex
2 − cos(3x)
x2
= lim
x→0
ex
2 − 1 + 1− cos(3x)
x2
= lim
x→0
ex
2 − 1
x2
+
1− cos(3x)
x2
=
= lim
x→0
ex
2 − 1
x2
+ lim
x→0
1− cos(3x)
x2
= lim
y→0
ey − 1
y
+ 9 lim
x→0
1− cos(3x)
(3x)2
=
= 1+9 lim
z→0
1− cos z
z2
= 1+9 lim
z→0
sen 2z
(1 + cos z)z2
= 1+9 lim
z→0
[ sen z
z
]2 1
(1 + cos z)
=
= 1 + 9
[
lim
z→0
sen z
z
]2
lim
z→0
1
(1 + cos z)
= 1 + 9 · 1 · 1
2
=
11
2
Pois limy→0
ey−1
y
= 1 e limz→0
sen z
z
= 1 são limites notáveis.
2. Considere a função
f(x) = arctan
[
3x+ 2
3x− 2
]
+
π
4
a) Prove que o gráfico da função f(x) passa pela origem (0, 0).
b) Calcule f ′(x)
c) Ache a equação da reta tangente ao gráfico da f(x) na origem (0, 0)
Solução a): É só verificar que f(0) = 0.
Com efeito f(0) = arctan
[
3·0+2
3·0−2
]
+ π
4
= arctan(−1) + π
4
= 0 [pois
arctan(−1) = −π
4
]
Solução b): Seja
u(x) = 3x+ 2 ; v(x) = 3x− 2 w(x) = u(x)
v(x)
Temos que
u′(x) = 3 v′(x) = 3
f ′(x) =
d
dx
[
arctan(w(x)) +
π
4
]
=
d
dx
arctan(w(x)) =
1
1 + w2(x)
dw(x)
dx
=
=
1
1 + w2(x)
d
dx
(
u(x)
v(x)
)
=
1
1 + w2(x)
×u
′(x)v(x)− u(x)v′(x)
v2(x)
=
=
3(3x− 2)− 3(3x+ 2)
(3x− 2)2 + (3x+ 2)
=
−12
18x2 + 8
=
−6
9x2 + 4
Solução c): A reta tangente procurada passa pela origem (0, 0) e tem
coeficiente angular m tal que m = f ′(x)|x=0 (i.e. m é igual ao o valor da
derivada da f(x) no ponto x = 0). Calculamos então f ′(x)|x=0.
f ′(x)|x=0 =
−6
9 · 0 + 4
=
−6
4
= −3
2
Logo é a reta que passa por (0, 0) e tem inclinação m = −3
2
, i.e.
a reta procurada tem equação y = −3
2
x.
3. Considere a função
f(x) =
√
x2 + 3x−
√
x2 − 1
a) Ache o domı́nio de f(x)
b) Encontre, caso existam, as asśıntotas horizontais do gráfico da f(x),
ou, caso o gráfico da f(x) não possua asśıntotas horizontais, justifique.
Solução a) Para que f(x) seja definido tem que ser x2 + 3x ≥ 0 e simul-
tanemanete x2 − 1 ≥ 0. Temos
x2+3x ≥ 0 ⇒ x(x+3) > 0 ⇒ [{x ≤ 0} ∩ {x ≤ −3}]
⋃
[{x ≤ 0} ∩ {x ≤ −3}]
i.e.
x2 + 3x ≥ 0 ⇒ (−∞,−3] ∪ [0,+∞)
e
x2 − 1 ≥ 0 ⇒ x2 ≥ 1 ⇒ [{x ≤ −1}]
⋃
[{x ≥ 1}]
i.e.
x2 − 1 ≥ 0 ⇒ (−∞,−1] ∪ [1,+∞)
Logo x2 + 3x ≥ 0 e simultaneamente x2 − 1 ≥ 0 se e somente se x ≤ −3
ou x ≥ 1. Logo o domı́nio Df da f(x) é
Df = (−∞,−3] ∪ [1,+∞) = R \ (−3,1)
Solução. b) Para encontrar as eventuais asśıntotas horizontais temos que
calcular limx→±∞ f(x). Temos
lim
x→+∞
f(x) = lim
x→∞
√
x2 + 3x−
√
x2 − 1 = lim
x→+∞
3x+ 1√
x2 + 3x+
√
x2 − 1
=
= lim
x→+∞
x
|x|
(3 + 1
x
)(√
1 + 3
x
+
√
1− 1
x2
) = lim
x→+∞
(3 + 1
x
)√
1 + 3
x
+
√
1− 1
x2
=
3
2
[Aqui usamos que limx→∞
x
|x| = 1]. Logo limx→∞ f(x) =
3
2
e logo
y = 3
2
é asśıntota horizontal do gráfico de f(x).
Por outro lado, procedendo como antes,
lim
x→−∞
f(x) = lim
x→−∞
x
|x|
(3 + 1
x
)(√
1 + 3
x
+
√
1− 1
x2
) = lim
x→−∞
−(3 + 1
x
)√
1 + 3
x
+
√
1− 1
x2
= −3
2
[Aqui usamos que limx→−∞
x
|x| = −1]. Logo limx→∞ f(x) = −
3
2
e portanto
y = −3
2
é também asśıntota horizontal do gráfico de f(x).
Em conclusâo:o gráfico da f(x) possui duas asśıntotas horizontais,
a saber, y = +3
2
e y = −3
2
.
prova 1 2013.1.pdf
Cálculo I: Gabarito 1a prova, Turmas R1, R2, OL. 13 de abril de 2013, 8h00, Duração: 1h40.
1. (7pts) Resolva 1x ≥
1
2−x .
Para começar, observe que é preciso considerar somente x 6= 0, x 6= 2. Colocando os termos do mesmo lado e colocando no
mesmo denominador, obtemos 2(x−1)x(x−2) ≥ 0 (1pts). Montando uma linha para cada termo que depende de x,
x
x − 1
x − 2
2(x−1)
x(x−2)
0 1 2
− 0 + + +
− − 0 + +
− − − 0 +
− + 0 − +
Portanto, o conjunto de soluções da desigualdade é S = (0, 1] ∪ (2,∞) (6pts).
Observação: Multiplicar os dois lados da desigualdade por x(2 − x) para obter a desigualdade 2 − x ≥ x é um erro, pois
não é verdade que x(2− x) ≥ 0 para todo x. Quem fez isso deve ter encontrado a solução errada (−∞, 1]. Neste caso demos
2 pontos.
2. (8pts) Determine todas as assíntotas de f(x) = x
2−7x+10
x2−2x . A reta x = 2 é assíntota vertical? Justi�que.
Procuremos primeiro assíntotas horizontais, calculando limites quando x tende a +∞ e −∞. Para começar, calculemos
limx→∞ f(x). Quando x toma valores grandes, os termos de grau maior são mais importantes. No caso, são os termos de
grau 2 que devem ser mais importantes. Portanto, coloquemos eles em evidência, e simpli�quemos:
lim
x→∞
f(x) = lim
x→∞
x2(1− 7x +
10
x2 )
x2(1− 2x )
= lim
x→∞
1− 7x +
10
x2
1− 2x
.
Como cada um dos termos 7/x, 10/x2 e 2/x tende a zero quando x → ∞, o numerador e o denominador ambos tendem a
1. Logo,
lim
x→∞
f(x) =
1
1
= 1 .(2.5pts)
Portanto, a reta horizontal de equação y = 1 é assíntota horizontal (a direita)(0.5pts). Mas quando x tende a −∞, são
também os termos de grau 2 que dominam, e a mesma colocação em evidência pode ser feita, e conclui-se que
lim
x→−∞
f(x) =
1
1
= 1 .
Portanto, a reta horizontal de equação y = 1 é assíntota horizontal (a esquerda). Finalmente, procuremos assíntotas
verticais. Para isso, os candidatos só podem vir de uma divisão por zero. Mas o denominador se anula se e somente se
x2 − 2x = x(x− 2) = 0. Assim, os candidatos são x = 0 e x = 2. Para determinar se tem de fato assíntotas verticais nesses
pontos, é preciso calcular limites laterais. Comecemos com
lim
x→0+
f(x) = lim
x→0+
1
x︸︷︷︸
tende a +∞
x2 − 7x+ 10
x− 2︸ ︷︷ ︸
tende a
10
−2=−5
Logo, limx→0+ f(x) = −∞ (1.5pts). Portanto, a reta vertical de equação x = 0 é assíntota vertical (0.5pts).
(Lembre-se que, para que haja uma assíntota vertical em um ponto, basta que um dos limites laterais nesse ponto seja igual
a ±∞. Assim, mesma conclusão pode ser obtida calculando-se o seguinte limite:
lim
x→0−
f(x) = lim
x→0−
1
x︸︷︷︸
tende a −∞
x2 − 7x+ 10
x− 2︸ ︷︷ ︸
tende a
10
−2=−5
Logo, limx→0− f(x) = +∞.)
Falta estudar os limites laterais em 2. Neste caso, o limite é da forma � 00 �, mas dá pra fatorar o numerador também, obtendo
lim
x→2
f(x) = lim
x→2
(x− 5)(x− 2)
x(x− 2)
= lim
x→2
x− 5
x
=
−3
2
= − 32 (2pts).
Logo, os dois limites laterais limx→2± f(x) existem e são �nitos (ambos valem 3/2!). Portanto, a reta vertical x = 2 não é
assíntota vertical (para ser, pelo menos um dos limites laterais x→ 2± deveria ser ±∞) (1.0pts).
3. (7pts) Se f(x) =
√
x+ 2, calcule limh→0
f(1+h)−f(1)
h .
Injetando os valores na função, o limite que se trata de calcular é
lim
h→0
f(1 + h)− f(1)
h
= lim
h→0
√
(1 + h) + 2−
√
3
h
(2pts) = lim
h→0
√
3 + h−
√
3
h
,
que é da forma indeterminada � 00 �. Multiplicando e dividindo pelo conjugado,
lim
h→0
√
3 + h−
√
3
h
= lim
h→0
(
√
3 + h−
√
3)(
√
3 + h+
√
3)
h(
√
3 + h+
√
3)
(2pts)
= lim
h→0
(
√
3 + h)2 −
√
3
2
)
h(
√
3 + h+
√
3)
= lim
h→0
1√
3 + h+
√
3
(2pts) =
1
2
√
3
(1pts) =
√
3
6
.
4. (8pts) Seja f(x) =

−2 se x ≤ 0 ,
x− 1 se 0 < x ≤ 2 ,
x2 − 2x+ 1 se x > 2 .
. f é contínua em x = 0? em x = 2? Justi�que, usando a de�nição de
continuidade.
1) Vamos calcular os limites de f(x) quando x → 0− e x → 0+. Para qualquer valor de x ≤ 0 temos que f(x) = −2,
assim limx→0− f(x) = −2. Por outro lado, para valores de x ≥ 0, mas próximo de 0, temos que f(x) = x − 1. Assim,
limx→0+ f(x) = limx→0+ x− 1 = −1. Como esses limites laterais calculados são distintos, a função não é contínua em x = 0
(4pts).
3) Vamos calcular os limites de f(x) quando x→ 2− e x→ 2+. Para valor esde x ≤ 2 próximo a 2, temos que f(x) = x− 1,
assim limx→2− f(x) = limx→2− x − 1 = 1. Por outro lado, para qualquer valores de x > 2 temos que f(x) = x2 − 2x + 1
assim, limx→2+ f(x) = limx→2+ x
2− 2x+1 = 22− 2 · 2+1 = 1. Como os limites laterais são iguais, temos que existe o limite
limx→2 f(x) = 1 (2pts). Como ainda f(2) = 1, temos que a função é contínua (2pts).
−2
−1
−2
5. (BONUS) Considere f : [−1, 0]→ C de�nida por f(x) = 2x +1. Determine C para que f seja uma bijeção, e dê a expressão
completa da sua função inversa.
A função f(x) = 2x + 1 é crescente e contínua. Como f(−1) = 3/2 e f(0) = 2, temos que a imagem do intervalo [−1, 0]
pela função f é igual ao intervalo [3/2, 2]. Além disso ela é uma bijeção entre esses dois intervalos. Assim, C = [3/2, 2] é o
conjunto C procurado. Vamos agora encontrar uma expressão para a função inversa f−1 : [3/2, 2]→ [−1, 0]. Para um dado
x ∈ [−1, 0] seja y ∈ [3/2, 2] tal que y = 2x + 1. Então temos que:
y = 2x + 1⇔ y − 1 = 2x ⇔ x = log2(y − 1).
Logo a função inversa é f−1(y) = log2(y − 1) (5pts).
2
Prova 1.png
Prova 2 - 2008.pdf
Universidade Federal de Minas Gerais
Instituto de Ciências Exatas – ICEx
Departamento de Matemática
Cálculo I
1o Semestre de 2008 – 2a Prova - 30/05/2008
1. (8 pontos) Calcule
(a)
∫
(3senx + 3x + 5)10(cos x + 1) dx.
Solução . Integrando por substituição:
u = 3senx + 3x + 5, du = (3 cosx + 3)dx, dx =
du
3(cos x + 1)
∫
(3senx+3x+5)10(cos x+1) dx =
∫
u10(cos x+1)
du
3(cosx + 1)
. =
1
3
∫
u10du =
1
3
(
u11
11
+ C) =
1
33
(3senx + 3x + 5)11 +
C
3
.
(b)
∫
3
√
8(t − 2)6(t + 1
2
)3 dt.
Solução .
∫
3
√
8(t − 2)6(t + 1
2
)3 dt =
∫
2(t − 2)2(t + 1
2
) dt =
∫
2(t2 − 4t + 4)(t + 1
2
) dt =
2
∫
(t3−4t2+4t+1
2
t2−2t+2) dt = 2
∫
(t3−7
2
t2+2t+2) dt =
∫
(2t3−7t2+4t+4) dt
= 2
t4
4
− 7t
3
3
+ 4
t2
2
+ 4t + C =
t4
2
− 7
3
t3 + 2t2 + 4t + C.
(c) lim
x→π
2
(
x
cotgx
− π
2 cosx
)
Solução . lim
x→π
2
(
xsenx
cos x
− π
2 cosx
)
= lim
x→π
2
(
2xsenx − π
2 cosx
)
(L’Hôpital) = lim
x→π
2
(
2senx + x cos x
−2senx
)
=
2
−2 = −1 .
(d) lim
x→0
x3/(x
4+ln x).
Solução . y = x3/(x
4+lnx) ⇒ ln y = 3
x4 + lnx
ln x.
L’Hôpital
=⇒ lim
x→0
ln y = lim
x→0
3 lnx
x4 + ln x
= lim
x→0
3
x
4x3 + 1
x
= lim
x→0
3
4x4 + 1
= 3.
⇒ lim
x→0
y = lim
x→0
eln y = e
lim
x→0
ln y
= e3.
2a questão. (8 pontos) Ache o domı́nio, máximos e mı́nimos, pontos de inflexão, conca-
vidade, asśıntotas e trace o gráfico de f(t) =
t2 + 9
(t − 3)2 .
Solução. (Usando o procedimento da aluna Júlia Hott para as derivadas)
Domı́nio: t 6= 3. Asśıntota vertical t = 3 :
lim
x→3
t2 + 9
(t − 3)2 = +∞
Asśıntota horizontal y = 1 :
lim
x→−∞
t2 + 9
(t − 3)2 = limx→−∞
1 + 9
t2
1 − 6
t
+ 9
t2
=
1
1
= 1 = lim
x→∞
t2 + 9
(t − 3)2 .
Derivadas: (Fazendo u=t-3)
f(t) =
t2 + 9
u2
⇒ f ′(t) = u
2(2t) − (t2 + 9)2uu′
u4
f ′(t) =
(t − 3)(2t) − 2(t2 + 9)u′
u3
=
u(2t) − 2(t2 + 9)
(t − 3)3 =
−6t − 18
(t − 3)3 =
−6t − 18
u3
.
f ′′(t) =
u3(−6) − (−6t − 18)3u2u′
u6
=
u(−6) − (−6t − 18)3uu′
u4
=
(t − 3)(−6) + (6t + 18)3
(t − 3)4
f ′′(t) =
12(t + 6)
(t − 3)4 .
Pontos cŕıticos: f ′(x) = 0 ⇔ −6t − 18 = 0 ⇔ t = −3
f ′′(−3) = 12(3)
(−6)4 =
1
36
> 0 ⇒ t = −3 é mı́nimo.
Concavidade: Temos que 12(t+6) junto com f ′′(t) mudam de sinal de negativo
para positivo em t = −6. Portanto f(t) é côncavo para baixo em (−∞,−6) ,
f(t) é côncavo para cima em (−6,∞) ⇒ t = −6 é ponto de inflexão de f(t).
VALORES DE f(t)
t −∞ −6 −3 0 3 4 +∞
f(t) 1 5/9 1/2 1 +∞ 25 1
questão. (8 pontos) Um fazendeiro deve cercar dois pastos retangulares, cada um deles com
lados a e b, com um lado comum a. Se cada pasto deve medir 400m2 de área,
determinar as dimensões a e b de maneira que o comprimento da cerca seja
mı́nimo.
Solução. C = comprimento da cerca, A = área de um pasto.
Função: C = 3a + 4b
Condição: A = ab = 400 ⇒ b = 400
a
C(a) = 3a +
1600
a, a > 0.
C ′(a) = 3 − 1600
a2
= 0 ⇒ 3 = 1600
a2
⇒ a2 = 1600
3
⇒ a = 40√
3
é ponto cŕıtico.
C ′′(a) =
3200
a3
> 0 ⇒ C ′′
(
400√
3
)
> 0 ⇒ a = 40√
3
é mı́nimo.
Resp: Dimensões: a =
40√
3
, b = 10
√
3.
2. 4a questão. (8 pontos)] Acumula-se areia em um monte com a forma de um
cone onde a altura é igual ao raio da base. Se o volume de areia cresce a
uma taxa de 10m3/h, a que razão aumenta a área da base quando a altura é
de 4m ?
Solução. r = raio do cone, V = volume do cone, A = área da base do cone.
V =
1
3
πr3 ⇒ r = 3
√
9πV 1/3
⇒ A = πr2 = π
(
3
√
3
π
V 1/3
)2
= 3
√
9πV 2/3
dA
dt
= 3
√
9π
(
2
3
)
V −1/3.
dV
dt
=
2
3
3
√
9π
V
.
dV
dt
Quando r = 4, V =
64π
3,
dV
dt
= 10 e
dA
dt
=
2
3
3
√
9π
64π
3
=
2
3
3
√
27
64
.10 =
2
3
3
4
.10 = 5
⇒ dA
dt
= 5m2/hora.
Prova 2 - 2011-1.pdf
Prova 2 - 2013.pdf
Cálculo I: Gabarito 2a prova, Turmas R1, R2, OL. 18 de maio de 2013, 8h00, Duração: 1h40.
1. (12pts) Uma corda de tamanho L é cortada em dois pedaços (não necessariamente iguais), e cada pedaço é usado para
fazer um quadrado. Qual é o jeito de cortar a corda que maximiza/minimiza a soma das áreas dos dois quadrados?
Suponha que o primeiro pedaço de corda tenha comprimento x ∈ [0, L]. Logo, a área do primeiro quadrado é igual a
(x/4)2. O segundo pedaço tem comprimento L−x, logo a área do segundo quadrado é igual a ((L−x)/4)2. Portanto,
a soma das áreas dos quadrados é dada por
A(x) = 116
(
x2 + (L− x)2
)
, x ∈ [0, L] .(3pts)
Procuremos os extremos globais de A em [0, L]. Primeiro procuremos os pontos críticos de A em (0, L). Como
A é obviamente derivável (polinômio de segundo grau em x), procuremos soluções de A′(x) = 0. Mas A′(x) =
1
16 (2x − 2(L − x)) =
1
8 (2x − L) (1pts). Logo, o único ponto crítico é x∗ = L/2 (1pts). O valor da área total nesse
ponto é A(x∗) =
1
162(L/2)
2 = L2/32 (1pts).
Considerando os valores de A na fronteira do intervalo: A(0) = 116L
2 e A(L) = 116L
2 (2pts). Logo, comparando com
o valor de A no ponto crítico, vemos que o mínimo global de A é atingido em x∗ e o máximo global em x = 0 e x = L.
Isto é, a área total é máxima quando a corda inteira é usada para criar um quadrado só, e a área mínima é obtida
cortando a corda em dois pedaços iguais (de tamanho L/2), e formando dois quadrados idênticos (cada um de lado
L/8) (4pts).
2. (15pts) Considere f(x) = (x2 − 3)e−x. Estude: o sinal, os zeros, as assíntotas (se tiver), a variação, e as posições dos
pontos de mínimos/máximos (locais, se tiver) no plano cartesiano. Em seguida, monte o grá�co detalhado.
Os zeros de f são −
√
3 e +
√
3. Como e−x > 0 para todo x e x2 − 3 = (x −
√
3)(x +
√
3), o sinal de f é dado por
(2pts)
f(x)
−
√
3 +
√
3
+ 0 − 0 +
Procurando as assíntotas ((2pts)): quando x → ∞, (x2 − 3)e−x = x
2−3
ex é da forma �
∞
∞ �, mas a Regra de Bernoulli-
l'Hôpital se aplica:
lim
x→∞
x2 − 3
ex
= lim
x→∞
2x
ex
= lim
x→∞
2
ex
= 0 .
Logo, y = 0 é assíntota horizontal (quando x → +∞). Quando x → −∞, o numerador de x
2−3
ex tende a +∞ e o
denominador é > 0 e tende a 0. Logo (a Regra de Bernoulli-l'Hôpital não se aplica, e) limx→−∞ f(x) = +∞ (não tem
assíntota horizontal quando x→ −∞).
A derivada se calcula da seguinte maneira:
f ′(x) = 2x · e−x + (x2 − 3) · (−e−x)
= −e−x(x+ 1)(x− 3) (3pts).
Logo, a variação de f é dada por:
x
f ′(x)
Var. de f
−1 3
− 0 + 0 −
min.min.
máx.máx.
Logo, f decresce em (−∞,−1], cresce em [−1, 3], e decresce em [3,∞), (−1, f(−1)) = (−1,−2e) é um ponto de
mínimo local, e (3, f(3)) = (3, 6e−3) é um ponto de máximo local (4pts). Finalmente, o grá�co é dado por
x
(x2 − 3)e−x
−
√
3 +
√
3
(−1,−2e)
(3, 6e−3)
(4pts)
Observe que o mínimo em (−1,−2e) é também global.
3. (8pts) Procure os intervalos em que a função f(x) = ln(1 + x2) é convexa/côncava.
Como a função é sempre bem de�nida e duas vezes derivavel, podemos estudar o sinal da sua segunda derivada. Mas
f ′(x) = 2x1+x2 (2pts), logo
f ′′(x) =
( 2x
1 + x2
)′
= 2
1 · (1 + x2)− x · (2x)
(1 + x2)2
= 2
1− x2
(1 + x2)2
.(2pts)
Como o denominador dessa fração é sempre > 0, o sinal de f ′′(x) é determinado pelo sinal de 1−x2. Portanto (2pts):
x
f ′′(x)
Conc.
de f
−1 +1
− 0 + 0 −
_ ^ _
Assim vemos que f é côncava em (−∞,−1], convexa em [−1, 1], e côncava em [1,+∞) (2pts). Observe que (−1, ln 2)
e (+1, ln 2) são pontos de in�exão.
4. (BONUS) Enuncie as hipóteses e as conclusões do Teorema de Rolle para uma função f : [a, b] → R com f(a) =
f(b) = 0. Em seguida, considere f(x) = (senx)2 no intervalo [0, π]. Caso as hipóteses do Teorema de Rolle sejam
veri�cadas, veri�que que a conclusão é verdadeira.
(O bônus vale (5pts).) Para o enunciado do Teorema de Rolle: apostila, Seção 5.5. A função f(x) = (senx)2 é
contínua em [0, π] e derivável em (0, π). Além disso, f(0) = f(π) = 0. Portanto, o Teorema de Rolle garante a
existência de pelo menos um ponto c ∈ (0, π) tal que f ′(c) = 0. No caso, temos f ′(x) = 2senx cosx, e vemos que
f ′(π2 ) = 0. Logo, c =
π
2 é um ponto em que a conclusão do teorema é verdadeira.
2
Prova 2 - 2013-1.pdf
Prova 2 - 2014.01 (2).png
Prova 2 - 2014.01.jpg
prova 3 2013.1.pdf
Cálculo I: Gabarito 3a prova, Turmas R1, R2, OL. 29 de junho de 2013, 8h00, Duração: 1h40.
1. (12pts) Calcule as primitivas:
∫
x3
x2−1 dx,
∫
xe3x dx.
O jeito mais simples de calcular a primeira é de fazer a substituição u = x2 − 1, que dá du = 2x dx, assim∫
x3
x2 − 1
dx =
∫
x2
x2 − 1
x dx = 12
∫
u+ 1
u
du = 12
∫ {
1 + 1u
}
du
= 12 {u+ ln |u|}+ C =
1
2
{
x2 − 1 + ln |x2 − 1|
}
+ C (6pts) .
Pode também considerar a função x
3
x2−1 como um caso
P (x)
Q(x) . Como o grau de P é maior que o grau de Q, comecemos
com uma divisão de polinômio, que dá x
3
x2−1 = x+
x
x2−1 (3pts). A primitiva do primeiro termo dá
∫
x dx = 12x
2+C1.
Para o segundo podemos chamar w = x2 − 1, dw = 2x dx, logo∫
x
x2 − 1
dx = 12
∫
1
w dw =
1
2 ln |w|+ C2 =
1
2 ln |x
2 − 1|+ C2(3pts)
A primitiva desse segundo termo pode ser calculada também fazendo uma decomposição em frações parciais, mesmo
se �ca um pouco mais complicado:∫
x
x2 − 1
dx = 12
∫ { 1
x− 1
+
1
x+ 1
}
dx = 12
{
ln |x− 1|+ ln |x+ 1|
}
+ C2 =
1
2 ln |x
2 − 1|+ C2 .(3pts)
Juntando com o primeiro termo obtemos de novo
∫
x3
x2−1 dx =
1
2
{
x2 + ln |x2 − 1|
}
+ C ′.
Para a segunda primitiva, basta usar integração por partes. Como 13e
3x é primitiva de e3x,∫
xe3x dx = x( 13e
3x)−
∫
1 · ( 13e
3x) dx (3pts)
= 13xe
3x − 13
∫
e3x dx
= 13xe
3x − 13 (
1
3e
3x) + C (3pts)
= 19e
3x(3x− 1) + C
2. Considere a região R �nita delimitada pela parábola f(x) = x2 − 7x+ 12 e pelo eixo x. (JUSTIFIQUE!)
(a) (5pts) Monte uma integral (sem calculá-la) que represente a área de R.
(b) (9pts) Monte uma integral (sem calculá-la) que represente o volume do sólido obtido girando R em torno da
reta 1) y = 0, 2) y = 2, 3) x = 5.
(a)
x
y = 2
x = 5
f(x)
3 4
R
Como a parábola corta o eixo x em a = 3 e b = 4, e está abaixo do eixo x entre esses dois valores, a área de R é dada
pela integral de Riemann (lembre da �área entre dois grá�cos�)∫ 4
3
(0− f(x))dx (5pts)
(b) Em torno da reta y = 0, usando cilíndros (pode também fazer com cascas mas �ca mais complicado):∫ 4
3
π(−f(x))2dx . (3pts)
Obs: formalmente, é preciso mudar o sinal de f(x) entre 3 e 4, pois o raio dos cilíndros usados na construção têm que
ser positivos!
Em torno da reta y = 2, usando também cilíndros:∫ 4
3
π(2− f(x))2dx−
∫ 4
3
π22dx . (3pts)
Obs: Os cilíndros externos têm raio dado pela diferença de altura entre a reta y = 2 e o grá�co da função f(x), que
dá 2− f(x). Os cilíndros internos têm todos o mesmo raio 2.
Em torno da reta x = 5, é mais fácil usando cascas:∫ 4
3
2π(5− x)(0− f(x)) dx . (3pts)
Obs: Uma casca tem raio dado pela distância do ponto x até a reta x = 5, que é 5− x; a sua altura é −f(x).
3. (8pts) De�na o que signi�ca uma integral imprópria
∫∞
a
f(x) dx convergir. A integral imprópria
∫∞
0
e−x
1+x2 dx converge?
Por de�nição, se f : [a,∞)→ R é contínua, diz-se que a integral imprópria
∫∞
a
f(x)dx converge se o limite
lim
L→∞
∫ L
a
f(x) dx
existir e for �nito (Obs: a integral
∫ L
a
f(x) dx é no sentido de Riemann) (4pts).
Para
∫∞
0
e−x
1+x2 dx, é preciso usar comparação (não dá para calcular a primitiva de
e−x
1+x2 ). Uma comparação possível é
de observar que para todo x ≥ 0, 0 ≤ e
−x
1+x2 ≤
e−x
1+0 = e
−x. Logo,
0 ≤
∫ ∞
0
e−x
1 + x2
dx ≤
∫ ∞
0
e−x dx = lim
L→∞
∫ L
0
e−x dx = lim
L→∞
{1− e−L} = 1 <∞ .
Logo
∫∞
0
e−x dx converge, portanto
∫∞
0
e−x
1+x2 dx converge também (4pts).
Um outro jeito: para todo x ≥ 0, 0 ≤ e−x ≤ 1, logo 0 ≤ e
−x
1+x2 ≤
1
1+x2 . Logo,∫ ∞
0
e−x
1 + x2
dx ≤
∫ ∞
0
1
1 + x2
dx = lim
L→∞
∫ L
0
dx
1 + x2
= lim
L→∞
arctanL = π2 <∞ .
Portanto,
∫∞
0
1
1+x2 dx converge, logo
∫∞
0
e−x
1+x2 dx converge também (4pts).
Obs: dá pra escrever também 0 ≤ e
−x
1+x2 ≤
1
x2 , só que a desigualdade para a integral imprópria,∫ ∞
0
e−x
1 + x2
dx ≤
∫ ∞
0
dx
x2
é inútil, já que a integral do lado direito é in�nita (o problema dela está em 0, não em →∞...)
4. (BONUS) Use uma substituição trigonométrica para calcular
∫
x3√
1−x2 dx.
Considere a substituição x = sent (dx = cos t dt). Como x é restrito ao intervalo −1 < x < 1, podemos supor que
−π/2 < t < π/2. Para esses valores de t temos que cos t > 0, logo
√
1− (sent)2 = | cos t| = cos t. Assim,∫
x3√
1− x2
dx =
∫
(sent)3√
1− (sent)2
cos t dt =
∫
(sent)3 dt (3pts) =
∫
(1− cos2 t)sent dt .
Com uma segunda substituição u = cos t (du = −sent dt),∫
(1− cos2 t)sent dt = −
∫
(1− u2) du = −u+
13u
3 + C = − cos t+ 13 (cos t)
3 + C .
Voltando para a variável x, ∫
x3√
1− x2
dx = −
√
1− x2 + 13
√
1− x2
3
+ C . (3pts)
2
prova suplementar 2014.1-solucao - Aldo.pdf
CÁLCULO I - SUPLEMENTAR
1. Considere a função
f(x) =
√
2x− x2
x
Calcule o volume do sólido de rotação obtido girando o gráfico da
f(x) em torno do seu asśıntota.
SOLUÇÃO A função f(x) não é definida em x = 0 e além disso
tem que haver que 2x− x2 ≥ 0, i.e. 0 ≤ x ≤ 2. Logo o domı́nio da
f(x) é o intervalo (0, 2]. Temos ainda que
lim
x→0+
f(x) = lim
x→0+
√
2x− x2
x
= lim
x→0+
√
2− x√
x
=
2
0+
= +∞
Logo o eixo y é assintota (vertical) para o gráfico da f , cuja projeção
no eixo x é o intervalo (0, 2]. Veja abaixo um esboço desse gráfico
com o eixo de rotação.
Rodando esse gráfico em volta do eixo y se obtem um volume ili-
mitado (mas finito, como veremos adiante) que pode ser calculado
por cascas ciĺındricas.
V = lim
�→0
∫ 2
�
2πxf(x)dx = 2π lim
�→0
∫ 2
�
√
2x− x2 =
= 2π
∫ 2
0
√
2x− x2 = 2π
∫ 2
0
√
1− (x− 1)2dx
Fazendo u = x− 1, du = dx e logo u ∈ [−1, 1]
V = 2π
∫ 1
−1
√
1− u2du = 4π
∫ 1
0
√
1− u2du
Fazendo u = sen θ, du = cos θdθ e logo θ ∈ [0, π/2]
V = 4π
∫ π
2
0
cos2 θdθ = 2π
∫ π
2
0
(1 + cos 2θ)dθ =
= 2π
[
θ +
sen 2θ
2
]π
2
0
= π2
3.Calcule a seguinte integral definida
I =
∫ π/2
0
(cosx)3( senx)2dx
SOLUÇÃO
I =
∫ π/2
0
(cosx)3( senx)2dx =
∫ π/2
0
cosx(1− ( senx)2)( senx)2dx =
=
∫ π/2
0
[( senx)2 − ( senx)4] cosxdx
Fazendo a substituição u = sen x, e logo du = cosxdx, e também
x ∈ [0, π/2] =⇒ u ∈ [0, 1] a integral vira
I =
∫ 1
0
[u2 − u4]du =
[
u3
3
− u
5
5
]1
0
=
1
3
− 1
5
=
2
15
3. Estude a função
f(x) = e
x2
2x−1
e esboçe seu gráfico.
SOLUÇÃO: Em x = 1/2 o denominador ao expoente se anula e
logo o domı́nio da f(x) é Df = R \ {1/2}. Temos que f(x) > 0 em
todo o domı́nio. Logo o gráfico da f vai estar todo acima do eixo x
e nunca cruza o eixo x. Temos que
lim
x→1/2+
e
x2
2x−1 = e
1/4
0+ = e+∞ = +∞ lim
x→1/2−
e
x2
2x−1 = e
1/4
0− = e−∞ = 0
Temos também que
lim
x→+∞
e
x2
2x−1 = lim
x→+∞
e
x
2−1/x = e+∞ = +∞
lim
x→−∞
e
x2
2x−1 = lim
x→−∞
e
x
2−1/x = e−∞ = 0
Logo y = 0 é assintota horizontal para f quando x→ −∞. Calcu-
lando a derivada primeira
f ′(x) =
2e
x2
2x−1x(x− 1)
(2x− 1)2
Logo f ′(x) = 0 em x = 1 e x = 0, f ′(x) não definida em x = 1/2.
Temos portanto que x = 1 e x = 0 são pontos cŕıticos para a f
(x = 1/2 não é ponto cŕıtico pois está fora do domı́nio da f).
Quanto ao sinal temos f(x) < 0 quando x ∈ (0, 1/2) ∪ (1/2, 1) e
f ′(x) > 0 quando x ∈ (−∞, 0) ∪ (1,+∞). Logo f é crescente em
(−∞, 0), passa a ser decrescente em (0, 1/2) e fica arbitrariamente
próxima de 0 quando x→ 1/2−. É arbitrariamente grande positiva
quando x → 1/2+, decresce em (1/2, 1), chegando ao valor e em
x = 1 e volta a crescer indefinitamente quando x ∈ (1,+∞). Logo
o ponto x = 0 é ponto de máximo local com f(0) = 1 e x = 1 é
ponto de mı́nimo local com f(1) = e. Observamos ainda que
lim
x→1/2−
f ′(x) = 0 lim
x→1/2+
f ′(x) = +∞
Logo o gráfico da f(x) chega com inclinação que tende a zero quando
x→ 1/2− e com inclinacão que tende a +∞ quando x→ 1/2+.
A derivada segunda é
f ′′(x) =
2e
x2
2x−1 [2x4 − 4x3 + 2x2 + 2x− 1]
(2x− 1)4
para entender onde se anula tem que achar as raizez do polinomio
p(x) = 2x4 − 4x3 + 2x2 + 2x− 1 = 2x2(x− 1)2 + 2x− 1
Não tendo p(x) fatoração simples procedemos da seguinte forma.
p(x) = 0⇐⇒ g(x) = h(x)
com
g(x) = 2x2(x− 1)2 h(x) = 1− 2x
Estudamos a equação g(x) = h(x) graficamente.
É imediato verificar que g(x) ≥ 0, g(x) tem dois mı́nimos em x = 0
e x = 1 onde g(1) = g(0) = 0 e tem um máximo para x = 1/2 onde
vale g(1/2) = 1/8 e limx→±∞ g(x) = +∞.
Por outro lado a função h(x) é a reta que passa pelos pontos (0, 1)
e (1/2, 0), logo a tradução gráfica da equação g(x) = h(x) cujas
soluções são as raizes de p(x) = 0 é a esboçada abaixo
onde se observa que g(x) = h(x) em dois pontos x = a ≈ 0.45
e x = b ≈ −0.65. Logo f ′′(x) = 0 nos pontos x = a ≈ 0.45 e
x = b ≈ −0.65 que são os pontos de inflexão do gráfico da f(x).
Veja abaixo um esboço do gráfico da f(x)

Teste o Premium para desbloquear

Aproveite todos os benefícios por 3 dias sem pagar! 😉
Já tem cadastro?

Outros materiais